LSAT and Law School Admissions Forum

Get expert LSAT preparation and law school admissions advice from PowerScore Test Preparation.

 cbchica
  • Posts: 5
  • Joined: Sep 06, 2016
|
#30198
Can someone please explain why D is the correct answer? I was between A and D, and I ended up choosing A.

Thanks,

cbchica
 Claire Horan
PowerScore Staff
  • PowerScore Staff
  • Posts: 408
  • Joined: Apr 18, 2016
|
#30220
(D) is the correct answer and could be pre-phrased before looking at the answer choices. This answer is correct because just because the trunk popped open due to other causes previously doesn't mean that, this time, the pothole wasn't the cause. Consider another example: You claimed that I laughed because you told me a particular joke. Previously I laughed without having heard that joke. Therefore, your joke didn't cause me to laugh. That would be a ridiculous conclusion, right?

Now, let's consider why (A) is wrong. (A) considers other cars and is outside the scope of the argument. Cynthia was making a claim about her car based on past events concerning the same car. For that reason, what other cars do is completely irrelevant.

For all LR questions, it is important to track the scope of the argument or question and to note any discrepancies in scope. Many, many wrong answer choices can be eliminated that way.
User avatar
 pineapplelover18
  • Posts: 8
  • Joined: Jun 01, 2024
|
#106784
hi everyone, I was stuck between C and D, and ultimately chose the right answer, but was wondering why C was wrong. Ray does Assume that if one event (the pothole) caused a certain event (no opening of the trunk) it could not cause a third event (trunk open), so why is this AC wrong? is it bc the trunk can not simultaneously be open and closed? so both effects (trunk closed, and third effect trunk open can not happen at once?)
 Luke Haqq
PowerScore Staff
  • PowerScore Staff
  • Posts: 795
  • Joined: Apr 26, 2012
|
#106872
Hi pineapplelover18!

I'm not fully sure I follow your explanation of Ray's presumption. Ray describes Cynthia as believing in a certain causal relationship--namely, the cause was the pothole, and the effect was that her trunk popped open.

Ray points out that the trunk had popped open on other occasions, and he reasons from this that it therefore could not have been a pothole that caused it to open on this occasion. As I understand (C), it would be saying something like Ray presumes, without providing justification, that if potholes cause the trunk to open, then it can't be the case that potholes cause a third event, such as the trunk not opening. But Ray doesn't make any presumption like that in the stimulus.

Get the most out of your LSAT Prep Plus subscription.

Analyze and track your performance with our Testing and Analytics Package.